Let f, g and h be the functions from the set of integers to the set of integers defined by f(x) = 2x +3, g(x) = 3x + 2 and h(x) = x3 +1.

(a) Find (fºg)(x) (b) Find (gof)(x) (c) l'ind (f)(x) (d) Find (h+h)(x) (e) Find h-1(x)

Answers

Answer 1

Let f, g and h be the functions from the set of integers to the set of integers defined by f(x) = 2x +3, g(x) = 3x + 2 and h(x) = x3 +1.

(a) To find (f º g)(x), we substitute g(x) into f(x) as follows:

  (f º g)(x) = f(g(x)) = f(3x + 2) = 2(3x + 2) + 3 = 6x + 4 + 3 = 6x + 9.

(b) To find (g º f)(x), we substitute f(x) into g(x) as follows:

  (g º f)(x) = g(f(x)) = g(2x + 3) = 3(2x + 3) + 2 = 6x + 9 + 2 = 6x + 11.

(c) To find the inverse of f(x), denoted as l'ind (f)(x), we solve for x in terms of f(x):

  x = (f(x) - 3) / 2.

  Rearranging the equation, we get f^(-1)(x) = 1/2x - 3/2.

(d) To find (h + h)(x), we add h(x) to itself:

  (h + h)(x) = h(x) + h(x) = ([tex]x^3[/tex] + 1) + (x^3 + 1) = 2[tex]x^3[/tex] + 2.

(e) To find the inverse of h(x), denoted as h^(-1)(x), we solve for x in terms of h(x):

  x = (h(x) - 1)^(1/3).

  Rearranging the equation, we get h^(-1)(x) = (x - 1)^(1/3).

Learn more about integers here:

https://brainly.com/question/490943

#SPJ11


Related Questions


(05.07 HC)
A student is assessing the correlation between the number of workers in a factory and the number of units produced daily?

Part A: Is there any correlation between the number of workers in a factory and the number of units produced daily? Justify your answer. (4 points)
Part B: Write a function that best fits the data. (3 points)
Part C: What does the slope and y-intercept of the plot indicate? (3 points)

Answers

Answer:

a) yes, because they both increase by the same increments each time. Tis can be represented by the equation y=5x+2

b) y=5x+2

c) The y-intercept represents the amount of units there were initially and the slope represents the amount of units for every worker.

Write an equation in point-slope form and slope-intercept form for each line
1. passes through (1, 9), slope = 2
2. passes through (4, -1), slope = -3

Answers

Answer:5.987

Step-by-step explanation:

if fis a differentiable function of rand g(x,y) = f(xy), show that x (dx)/(dg) - y (dg)/(dy) = 0

Answers

To prove that x(dx/dg) - y(dg/dy) = 0, we'll start by finding the derivatives of the functions involved.

Given that g(x, y) = f(xy), we can find the partial derivatives of g with respect to x and y using the chain rule:

∂g/∂x = ∂f/∂u * ∂(xy)/∂x = y * ∂f/∂u

∂g/∂y = ∂f/∂u * ∂(xy)/∂y = x * ∂f/∂u

Now, let's differentiate the equation x(dx/dg) - y(dg/dy) = 0:

d/dg (x(dx/dg) - y(dg/dy)) = d/dg (x(dx/dg)) - d/dg (y(dg/dy))

Using the chain rule, we can rewrite the derivatives:

d/dg (x(dx/dg)) = d/dx (x(dx/dg)) * dx/dg = x * d/dx (dx/dg)

d/dg (y(dg/dy)) = d/dy (y(dg/dy)) * dg/dy = y * d/dy (dg/dy)

Substituting these expressions back into the equation, we have:

x * d/dx (dx/dg) - y * d/dy (dg/dy) = 0

Now, let's simplify the equation further. Since dx/dg represents the derivative of x with respect to g, it is essentially the reciprocal of dg/dx, which represents the derivative of g with respect to x:

dx/dg = 1 / (dg/dx)

Similarly, dg/dy represents the derivative of g with respect to y. Therefore, we can rewrite the equation as:

x * d/dx (1/(dg/dx)) - y * d/dy (dg/dy) = 0

Taking the derivatives with respect to x and y, we have:

[tex]x * (-1/(dg/dx)^2) * (d^2g/dx^2) - y * (d^2g/dy^2) = 0[/tex]

Since dg/dx and dg/dy are partial derivatives of g, we can simplify further:

x * (-1/(∂g/∂x)^2) * (∂^2g/∂x^2) - y * (∂^2g/∂y^2) = 0

Finally, using the expressions we found for the partial derivatives of g earlier, we can substitute them into the equation:

x * (-1/(y * ∂f/∂u)^2) * (∂^2f/∂u^2 [tex]* y^2[/tex]) - y * (∂^2f/∂u^2 * [tex]x^2[/tex]) = 0

Canceling out the common factors, we are left with:

∂^2f/∂u^2 * x + ∂^2f/∂u^2 * y = 0

Since ∂^2f/∂u^2 is a constant (it does not depend on x or y), we can factor it out:

∂^2f/∂u^2 * (y - x) = 0

For the equation to hold, we must have either ∂^2f/∂u^2 = 0 or (y - x) = 0. However, the second condition (y - x) = 0 implies that y = x, which is not a necessary condition for the given equation to be true.

Therefore, the only possibility is ∂^2f/∂u^2 = 0, which implies that the equation x(dx/dg) - y(dg/dy) = 0 holds.

In conclusion, we have shown that x(dx/dg) - y(dg/dy) = 0 under the assumption that f is a differentiable function of r and g(x, y) = f(xy).

Learn more about differnetiation here:

https://brainly.com/question/28099315

#SPJ11

Rolling a single six-sided di, you play a game with the following rules: if you roll an even number, you lose 1 point. If you roll a 1, you gain 1 point. If you roll a 3, you gain 3 points. If you roll a 5, you lose 4 points. After a long time continually playing the game, would you expect to have a positive point total or a negative point total?

Answers

The expected value is 0, which means that, on average, you neither gain nor lose points over the long run. This suggests that after playing the game for a long time, we would expect to have a point total close to zero.

To determine whether you would expect to have a positive or negative point total after a long time playing the game, we can calculate the expected value or average point gain/loss per roll.

Let's calculate the expected value for each outcome:

Rolling an even number:

Probability = 3/6 = 1/2,

Point gain/loss = -1

Rolling a 1:

Probability = 1/6,

Point gain/loss = 1

Rolling a 3:

Probability = 1/6,

Point gain/loss = 3

Rolling a 5:

Probability = 1/6,

Point gain/loss = -4

The expected value, we multiply each outcome's point gain/loss by its probability and sum them up

Expected Value = (1/2) × (-1) + (1/6) × 1 + (1/6) × 3 + (1/6) × (-4)

Expected Value = -1/2 + 1/6 + 1/2 - 2/3

Expected Value = 0

The expected value is 0, which means that, on average, you neither gain nor lose points over the long run. This suggests that after playing the game for a long time, you would expect to have a point total close to zero.

To know more about expected value click here :

https://brainly.com/question/29068283

#SPJ4

Use the Division Algorithm to show that the cube of any integer is of the form 9k, 9k + 1 or 9k + 8. (Hint: By the Division Algorithm, the integer a is of one of the forms 9q, 9q + 1, ..., 9q + 8; establish the result for 9q + 3 and 9q + 7 only.)

Answers

Using the Division Algorithm we have shown that the cube of any integer is of the form 9k, 9k + 1 or 9k + 8

To show that the cube of any integer is of the form 9k, 9k + 1, or 9k + 8, we will use the Division Algorithm. We will establish the result for the cases of 9q + 3 and 9q + 7.

Let's consider the case of 9q + 3, where q is an integer. We want to show that the cube of any integer of the form 9q + 3 is also of the form 9k, 9k + 1, or 9k + 8.

Let's choose an arbitrary integer, let's say n, such that n = 9q + 3.

Taking the cube of n:

n³ = (9q + 3)³

    = 729q³ + 243q² + 27q + 27

Now, let's express this in terms of 9k, 9k + 1, or 9k + 8:

n³ = 729q³ + 243q² + 27q + 27

    = 9(81q³ + 27q² + 3q + 3) + 18 + 9

    = 9(81q³ + 27q² + 3q + 3 + 2) + 7

We can see that n³ can be expressed in the form 9k + 7, where k = 81q³ + 27q² + 3q + 3 + 2.

Therefore, we have shown that for the case of 9q + 3, the cube of any integer of that form is of the form 9k + 7.

Now, let's consider the case of 9q + 7, where q is an integer. We want to show that the cube of any integer of the form 9q + 7 is also of the form 9k, 9k + 1, or 9k + 8.

Similar to the previous case, let's choose an arbitrary integer, let's say n, such that n = 9q + 7.

Taking the cube of n:

n³ = (9q + 7)³

    = 729q³ + 441q² + 147q + 49

Now, let's express this in terms of 9k, 9k + 1, or 9k + 8:

n³ = 729q³ + 441q² + 147q + 49

    = 9(81q³ + 49q² + 16q + 5) + 4

We can see that n³ can be expressed in the form 9k + 4, where k = 81q³ + 49q² + 16q + 5.

Therefore, we have shown that for the case of 9q + 7, the cube of any integer of that form is of the form 9k + 4.

By using the Division Algorithm and establishing the results for the cases of 9q + 3 and 9q + 7, we have shown that the cube of any integer is of the form 9k, 9k + 1, or 9k + 8.

Learn more about Division Algorithm here

https://brainly.com/question/14542601

#SPJ4

The expression 2 3 4 2 is equivalent to A. 2 7 B. 2 12 C. 8 5 D. 8 6

Answers

Answer:

B. 2 12

Step-by-step explanation:

maaf kalo salah ituh jawabbanya menurut guwah

7th grade math!
Can somebody plz help answer these questions correctly (only if you remmeber how to do this) thx so much :3

WILL MARK BRAINLIEST WHOEVER ANSWERS FIRST :DD

Answers

Answer:

I just learned about this, and if its wrong i am so so sorry

Step-by-step explanation:

a= 120

b= 60

w= 120

x= 120

r= 105

q= 105

p= 105

Answer:

a= 120

b= 60

w= 120

x= 120

r= 75

q= 105

p= 105

Step-by-step explanation:

what is the best approximation for the area of this circle? use 3.14 to approximate pi. responses
a.12.6 m²
b.25.1 m²
c.50.2 m²
d.158.0 m²

Answers

The best approximation for the area of this circle is approximately 50.2 m².Option (c) is the correct answer.

To determine the best approximation for the area of this circle, we need to use the formula for the area of a circle, which is given by A = πr².

Here, we are given the value of π to be approximately equal to 3.14.

Now, we need to determine the radius of the circle.

From the diagram, we can see that the diameter of the circle is 8 meters.

Therefore, the radius is half of this, which is 4 meters.

Substituting the values of π and r into the formula, we get: A = πr²= 3.14 × 4²= 3.14 × 16= 50.24 (to two decimal places)

Therefore, the best approximation for the area of this circle is approximately 50.2 m².Option (c) is the correct answer.

know more about area of this circle

https://brainly.com/question/28642423

#SPJ11

1. This Question Is Compulsory (a) Find the following Laplace transform and verse Laplace transform (1) Llet+')} (1) L- [8 marks) (b) 18 = 1+k8-2+ 3+ 3k and 7 - 1 - 2k, obtain the following (1) ä. ax, (1) 6x5)-(ä xk), (iv) 2:a)3 +34 (12 marks) () Find the point at which the plane 22 - 5y + z = 5 and the line Ft) - (+1) + (24+ 1)3+ (t+1)! (where t is a real number) intersect. [3 marks) (d) Compute the curvature and principal unit normal vector for the curve rt) 2 sin(t)+ 2 con(e) { for t > 0. [6 marks] 2t? (e) For the two matrices A= 12 0 0-7 5 3 B= B=(- :) 0 Find (1) AT, (1) BT, (ii) B(AT) and (iv) (AB)". [8 marks) (t) Find the determinant and trace of the matrix 5-13 10 0 - 2 1 0 3 [6 marks) Solve the following system of simultaneous equations using Gauss-Jordan elimination: 2.11 + 12 = -2, -92+372 = 4. [7 marks] Page 2 of 4

Answers

Part a. Find the Laplace transform and inverse Laplace transform of Let+')} and L- [8 marks]Laplace transform of Let+')} is given as: L(et+')} = 1/s-(1/ s+2)Let's try to  this.1/s-(1/ s+2) = (s+2-s)/s(s+2) = 2/s(s+2)L-1{2/s(s+2)}= L-1{(1/s)-(1/s+2)} = e^(-2t) - 1

Part b. 18 = 1+k8-2+ 3+ 3k and 7 - 1 - 2k, obtain the following: i) ä. ax, ii) (ä xk), iii) 2:a)3 +34 i) 18 = 1+k8-2+ 3+ 3k

Let's simplify this as follows.18 = 12 + k + 3k - 2k + 3k-1 - 4k+1/218 = 12 - k + 2k + 2k + 3(1/k) - 4k+1/2a=12, b=-1, c=2 and d=2Therefore,ä. ax = ad-bc = 24 + 2 = 26ii) (ä xk) = (cd - bc)i + (ab - ad)j + (ad - bc)k = 1i - 2j + 24kiii) 2:a)3 +34 = 2(ad-bc) + 3(ab-ad) + 4(cd-bc) + 3(bd - ac) = 52
Part c. Find the point at which the plane 22 - 5y + z = 5 and the line Ft) - (+1) + (24+ 1)3+ (t+1)! (where t is a real number) intersect. [3 marks]. Let's substitute the given line in the plane equation. 2(1+4t) - 5(3+1t) + z = 5 solving for z, we get

z = 12t - 13

Substitute this z in line equation to get the point of intersection.(1+t, 4+2t, 12t-13)

Part d. Compute the curvature and principal unit normal vector for the curve rt) 2 sin(t)+ 2 con(e) { for t > 0. [6 marks]. Given curve r(t) = 2 sin(t) + 2 cos(t).

We need to find the first and second derivatives. r'(t) = 2 cos(t) - 2 sin(t)r''(t) = -2cos(t) - 2sin(t)

From these values, we get |r'(t)| = sqrt(8)K(t) = |r'(t)|/|r"(t)|^3/2K(t) = 2^(3/2)/8^(3/2)K(t) = 1/4^(1/2)K(t) = 1/2

Therefore the curvature is 1/2Now, let's find the principal unit normal vector. N(t) = r''(t)/|r"(t)|N(t) = <-1/sqrt(2),1/sqrt(2)>
Part e. For the two matrices A= 12 0 0-7 5 3 B= B=(- :) 0

Find i) AT, ii) BT, iii) B(AT) and iv) (AB)" [8 marks]

i) AT =Transpose of A = 1 -7 0 2 5 3

ii) BT =Transpose of B = 1 0 -3 0 2 1

iii) B(AT) =B(Transpose of A) = 1 -7 0 2 5 3(-1) 0 0 7 -5 -3= -1 7 0 -2 -5 -3

iv) (AB)" =(AB)^-1=Inverse of AB Let's calculate AB first. AB =12 0 0-7 5 3(-1) 0 0-5 2 -1= -1 0 0 1

Therefore, the inverse of AB is1 0 0-1

To know more about Laplace transform refer to:

https://brainly.com/question/31583797

#SPJ11

PLEASEEEEEEEEEEEEEEE HELPPPPPPPPPPPPPPPPPPPPPPP

Answers

Dont press that persons link! Ok im not sure if this is the correct answer, but anyways.

85 + x = 131
-85. -85
x= 46

A national health survey weighed a sample of 490 boys aged 6-11 and found that 67 of them were overweight. They weighed a sample of 530 girls aged 6-11 and found that 66 of them were overweight Conduct a hypothesis test to determine whether the proportion of overweight kids aged 6-11 among boys is greater than the proportion of overweight kids aged 6-11 among girls? Use level of significance 10%.

Answers

As the lower bound of the 90% confidence interval is below 0, there is not enough evidence to conclude that  the proportion of overweight kids aged 6-11 among boys is greater than the proportion of overweight kids aged 6-11 among girls.

How to obtain the confidence interval?

The sample proportions are given as follows:

Boys: 67/490 = 0.1367. Girls: 66/530 = 0.1245.

The difference is then given as follows:

0.1367 - 0.1245 = 0.0122.

The standard error for each sample is given as follows:

[tex]s_B = \sqrt{\frac{0.1367(0.8633)}{490}} = 0.0153[/tex][tex]s_G = \sqrt{\frac{0.1285(0.8715)}{530}} = 0.0145[/tex]

Then the standard error for the distribution of differences is given as follows:

[tex]s = \sqrt{0.0153^2 + 0.0145^2}[/tex]

s = 0.0211.

The critical value for a 90% confidence interval is given as follows:

z = 1.645.

The lower bound of the interval is:

0.0122 - 1.645 x 0.0211 = -0.0225.

More can be learned about the z-distribution at https://brainly.com/question/25890103

#SPJ4

A sample of 92 one-year-old spotted flounder had a mean length of 123.47 millimeters with a sample standard deviation of 18.72 millimeters, and a sample of 138 two-year-old spotted flounder had a mean length of 129.96 millimeters with a sample standard deviation of 31.60 millimeters. Construct an 80% confidence interval for the mean length difference between two-year-old founder and one-year-old flounder. Let µ_1, denote the mean tength of two-year-old flounder and round the answers to at least two decimal places.
An 80% confidence interval for the mean length difference, in millimeters, between two-year-old founder and one-year old flounder is___<µ_1-µ_2<_____

Answers

The 80% confidence interval for the mean length difference between two-year-old and one-year-old spotted flounder is [5.15, 7.83] millimeters.

How to calculate the value

For an 80% confidence interval, we need to find the critical value associated with a two-tailed test. Since the sample sizes are large, we can use the z-distribution. The critical value for an 80% confidence interval is approximately 1.282.

ME = 1.282 * 1.0466 ≈ 1.3426

Confidence interval = Point estimate ± Margin of error

Confidence interval = 6.49 ± 1.3426

Finally, rounding to at least two decimal places:

Confidence interval = [5.15, 7.83]

Therefore, the 80% confidence interval for the mean length difference between two-year-old and one-year-old spotted flounder is [5.15, 7.83] millimeters.

Learn more about confidence interval on

https://brainly.com/question/15712887

#SPJ4

Cranor and Christensen study diabetics insured by two employers. Group 1 subjects were employed by the City of Asheville, North Carolina, and group 2 subjects were employed by Mission-St. Joseph’s Health System. The data are displayed in the following table.

Weight (Pounds) Group 1 Group 2 252 215 240 185 195 240 190 302 310 210 205 270 312 212 190 200 159 126 238 172 170 204 268 184 190 170 215 215 136 140 320 254 183 200 280 148 164 287 270 264 214 288 210 200 270 270 138 225 212 210 265 240 258 182 192 203 217 221 225 126 Source: Data provided courtesy of Carole W. Carnor, Ph.D. 220 295 202 268 220 311 164 206 170 190

ANSWER SHOULD BE BASE ON THE FF:
State the null and alternative hypothesis
Level of significance
Test statistics
Decision Rule
Stata Output
Statistical Decision
Conclusion
Linear Regression Model and Interpretation (if necessary)

Answers

Null and alternative hypothesis: The null hypothesis is that the mean weight of the diabetics insured by two employers is similar.

In contrast, the alternative hypothesis is that the mean weight of the diabetics insured by two employers is not equal. Level of significance: The level of significance

(α) is the probability of rejecting the null hypothesis when it is valid. The commonly used level of significance is 0.05.

Test statistic: Assuming the population variances are equal, the test statistic is the t-distribution.

Decision Rule: Reject H0 if the t-value is greater than 2.060 or less than -2.060 and accept H0 if the t-value is between 2.060 and -2.060.

State Output: Assuming equal variances, the p-value associated with a two-tailed t-test for equality of means is 0.2682.Statistical decision: Since the p-value is greater than 0.05, we accept the null hypothesis and conclude that the mean weight of diabetics insured by the two employers is equal.

Conclusion: Therefore, there is no considerable difference in the mean weight of diabetics insured by two employers in the City of Asheville and Mission-St. Joseph’s Health System. Linear regression model and interpretation (if necessary): A linear regression model was not necessary in this study.

To know more about probability refer to:

https://brainly.com/question/27342429

#SPJ11

Right will be marked brainlist

Answers

Answer:

the answer is 3,107.21

The answer to that question is that one

Zc and Zx are supplementary angles. Zy measures 49º.
What is the measure of Zx?

Answers

Answer:

Zx = 131

Step-by-step explanation:

The angles will equal to 180, so therefore you will do 180-49=Zx

Boilermaker House Painting Company incurs the following transactions for September.
1. September 3 Paint houses in the current month for $16,000 on account.
2. September 8 Purchase painting equipment for $17,000 cash.
3. September 12 Purchase office supplies on account for $2,700.
4. September 15 Pay employee salaries of $3,400 for the current month.
5. September 19 Purchase advertising to appear in the current month for $1,100 cash.
6. September 22 Pay office rent of $4,600 for the current month.
7. September 26 Receive $11,000 from customers in (1) above.
8. September 30 Receive cash of $5,200 in advance from a customer who plans to have his house painted in the following month.
BOILERMAKER HOUSE PAINTING COMPANY
Trial Balance
Accounts Debit Credit
Cash $11,300selected answer incorrect not attempted
Accounts Receivable 6,200selected answer incorrect not attempted
Supplies 2,900selected answer incorrect not attempted
Equipment 22,400selected answer incorrect not attempted
Accounts Payable not attempted 3,600selected answer incorrect
Deferred Revenue not attempted 5,000selected answer incorrect
Common Stock not attempted 20,000selected answer incorrect
Retained Earnings not attempted 8,000selected answer incorrect
Service Revenue not attempted 15,000selected answer incorrect
Salaries Expense 3,200selected answer incorrect not attempted
Advertising Expense 1,200selected answer incorrect not attempted
Rent Expense 4,400selected answer incorrect not attempted
Totals $51,600 $51,600

Answers

Boilermaker House Painting Company's trial balance reflects a debit balance of $11,300 in Cash, $6,200 in Accounts Receivable, and $2,900 in Supplies.The credit side shows $22,400 in Equipment, $3,600 in Accounts Payable, $5,000 in Deferred Revenue, $20,000 in Common Stock, $8,000 in Retained Earnings, $15,000 in Service Revenue, $3,200 in Salaries Expense, $1,200 in Advertising Expense, and $4,400 in Rent Expense.

The trial balance provides a summary of Boilermaker House Painting Company's financial transactions for the month of September. The company engaged in several activities during the month, including providing painting services, purchasing equipment and office supplies, paying salaries, renting office space, and receiving cash from customers.

In the first transaction, the company painted houses for customers in the current month, generating service revenue of $16,000 on account. This resulted in an increase in the Accounts Receivable balance, representing the amount owed by customers.

The second transaction involved the purchase of painting equipment for $17,000 in cash. This expenditure was recorded as an increase in the Equipment account, which reflects the company's tangible assets.

Next, the company purchased office supplies on account for $2,700. This transaction increased the Supplies account and created an obligation in the form of an Accounts Payable.

The fourth transaction involved paying employee salaries of $3,400 for the current month. This expense was recorded in the Salaries Expense account, which represents the cost of labor incurred by the company.

In the fifth transaction, the company spent $1,100 in cash to purchase advertising, which was intended to appear in the current month. This expense was recorded in the Advertising Expense account.

The sixth transaction involved paying office rent of $4,600 for the current month. This expense was recorded in the Rent Expense account, representing the cost of utilizing office space.

In the seventh transaction, the company received $11,000 in cash from customers who had previously been billed for the painting services provided. This increased the Cash balance, reflecting the inflow of funds.

Lastly, the company received $5,200 in advance cash from a customer who planned to have their house painted in the following month. This created a liability in the form of Deferred Revenue, as the company had not yet provided the corresponding service.

Learn more about the Accounts:
brainly.com/question/14138124

#SPJ11

An incoming college student took her college’s placement exams in French and mathematics. In French, she scored 85 and in math 80. The overall results for both exams are approximately normal. The French mean score was 72 with a standard deviation of 12, while the mean math score was 70, with a standard deviation of 7.8. On which exam did she do better compare with the other incoming college students? Compute the z-scores (round to 2 decimal places) and the percentiles (round to the nearest whole) for each exam to support your answer.

Answers

Answer:

Following are the responses to the given question:

Step-by-step explanation:

[tex]French \ \ \ \ \ \ \ \ \ \ \ \ \ \ \ \ \ \ \ \ \ \ \ \ \ \ \ \ \ \ \ \ \ \ Mathematics\\\\X= 85\ \ \ \ \ \ \ \ \ \ \ \ \ \ \ \ \ \ \ \ \ \ \ \ \ \ \ \ \ \ \ \ \ \ X= 80\\\\\mu=72\ \ \ \ \ \ \ \ \ \ \ \ \ \ \ \ \ \ \ \ \ \ \ \ \ \ \ \ \ \ \ \ \ \ \mu =70\\\\\sigma=12 \ \ \ \ \ \ \ \ \ \ \ \ \ \ \ \ \ \ \ \ \ \ \ \ \ \ \ \ \ \ \ \ \ \ \sigma= 7.8\\\\[/tex]

Formula:

[tex]\bold{Z-Score=\frac{x-\mu}{\sigma}}\\\\French \ \ \ \ \ \ \ \ \ \ \ \ \ \ \ \ \ \ \ \ \ \ \ \ \ \ \ \ \ \ \ \ \ \ Mathematics\\\\Z=\frac{85-72}{12} \ \ \ \ \ \ \ \ \ \ \ \ \ \ \ \ \ \ \ \ \ \ \ \ \ \ \ \ \ \ \ \ \ \ Z=\frac{80-70}{7.8}\\\\=1.0833 \ \ \ \ \ \ \ \ \ \ \ \ \ \ \ \ \ \ \ \ \ \ \ \ \ \ \ \ \ \ \ \ \ \ \ \ \ \ =1.2820\\\\[/tex]

In the french exam Z value is 1.0833 and in the maths exam Z value is 1.2820 that's why we can say that  in maths exam, she does better than french exam.

|x+8|=x+8 what is x?

Answers

Answer:

x ≥ 0

Step-by-step explanation:

The absolute value of x+8 equals itself, thus x must be a real number (0 or any positive number).

The value of x in |x+8|=x+8 is -8.

What is an equation?

An equation is an expression that shows the relationship between two or more numbers and variables.

A mathematical equation is a statement with two equal sides and an equal sign in between. An equation is, for instance, 4 + 6 = 10. Both 4 + 6 and 10 can be seen on the left and right sides of the equal sign, respectively.

We are given that;

|x+8|=x+8

Now,

To solve this equation, we need to consider two cases:

Case 1: x+8 is positive or zero. Then |x+8|=x+8 and we can solve for x by subtracting 8 from both sides:

|x+8|=x+8

x+8=x+8

x=x

This means that any value of x is a solution in this case.

Case 2: x+8 is negative. Then |x+8|=-(x+8) and we can solve for x by adding 8 to both sides and dividing by -2:

|x+8|=-(x+8)

x+8=-(x+8)

2x=-16

x=-8

Therefore, by the given equation the answer will be -8.

Learn more about equations here;

https://brainly.com/question/25180086

#SPJ2

Express 2^6 x (1/4)^5 / (16)^3 as a power with a base of 4

Answers

the expression 2⁶ × (1/4)⁵ / (16)³ can be written as 4⁻⁸.

To express the given expression 2⁶ × (1/4)⁵ / (16)³ as a power with a base of 4, we can simplify the expression using the properties of exponents:

2⁶ × (1/4)⁵ / (16)³

First, we simplify the exponents:

2⁶ = 64 = 4³

(1/4)⁵ = 4⁻⁵

(16)³ = 4⁶

Now, we substitute these simplified values back into the expression:

4³ × 4⁻⁵/4⁶ = 4³ × 4⁻⁵ × 4⁻⁶

= 4³⁻⁵⁻⁶

= 4⁻⁸

Finally, we express the simplified expression as a power with a base of 4: 4⁻⁸

Therefore, the expression 2⁶ × (1/4)⁵ / (16)³ can be written as 4⁻⁸.

Learn more about expression here

https://brainly.com/question/20511436

#SPJ4

Can y’all Plzs hep me I need this

Answers

Use Pythagorean theorem
a^2 + b^2 = c^2
In other words, sum of the squared smaller lengths are equal to the squared of the longest length

15^2 + 17^2 = c^2
225 + 289 = c^2
c^2 = 514
c = square root of 514
= 22.67156
Round it

Solution: 22.7 km

Answer:

22.7 km

Step-by-step explanation:

The Pythagorean Theorem is:

a² + b² = c²

where a and b are legs and c is the hypotenuse.

The hypotenuse is the longest side of a right triangle and can be identified as being across from the right angle. The legs are interchangeable between the variables a and b.

Because it's clear that x is the hypotenuse:

15² + 17² = x²

225 + 289 = x²

514 = x²

√514 = √x²

x = √514

Because we can't simplify √514 anymore, we can keep it as radical 514 or round it:

22.6715681 ≈ 22.7

A bag contains three different balls, one red (r), one blue(b), one white(w). TWO balls are drawn from the bag without replacement one after the other (at random without looking) and the colors recorded. This means once the first color is drawn, the first ball is kept out of the bag and only 2 colors remain when the second ball is drawn. List the sample space. [Use lower case letters for the colors, preserve the given order, commas separating pairs, no spaces.]

Answers

Answer:

The total sample list is 6

Step-by-step explanation:

The bag has following balls

Red - 1

Blue -1

White -1

Two balls are drawn from the bag without replacing the other -

The probability of drawing 1st ball of any color - 1/3

The probability of drawing 2nd ball of any color - 1/2

These two events are independent of each other

Hence, the probability of deriving two balls without replacement is 1/3*1/2 = 1/6

Hence, the total sample list is 6

The sample space is the list of possible outcomes of an experiment

The sample space is {rb, rw, br, bw, wr, wb}

The color of the three balls is represented as:

Red = r

Blue = b

White = w

Given that the selection is without replacement, the sample space would be:

rb, rw, br, bw, wr, wb

The count of the sample space represents the sample size.

Hence, the sample size of the experiment is 6, and the sample space is {rb, rw, br, bw, wr, wb}

Read more about sample space at:

https://brainly.com/question/16347135

I’m begging you please please ASAP ASAP please ASAP thank you please please ASAP

Answers

Answer:

The ratio is [tex]\frac{2}{3}[/tex].

Step-by-step explanation:

The ratio of the perimeters of Quad ABCD to Quad WXYZ = [tex]\frac{perimeter of ABCD}{perimeter of WXYZ}[/tex]

But considering sides AB and WX,

representative factor for both figures = [tex]\frac{12}{8}[/tex]

So that;

WX = 12

XY = 1.5 x 6 = 9

YZ = 1.5 x 7 = 10.5

WZ = 1.5 x 7 = 10.5

Thus,

perimeter of Quad ABCD = 6 + 7 + 7 + 8

                                           = 28

perimeter of Quad WXYZ = 9 + 10.5 + 10.5 + 12

                                           = 42

The ratio of the perimeters of Quad ABCD to Quad WXYZ = [tex]\frac{28}{42}[/tex]

                                                         = [tex]\frac{2}{3}[/tex]

Of the 1500 students at Marshall
Junior High, 38% are 7 graders.
What is the total number of 7
graders at Marshall Junior High?
Write a proportion and solve.

Answers

Answer:

36+25= 61

61/600 x 100 = 10.1 %

Step-by-step explanation:   Hope This helped!!!!

Help ASAP! Will name brainliest!

Answers

Q 9 the answer is 250

Find all solutions of the equation in the interval [0°, 360°). cos(x) = -1 ​

Answers

Answer:

180°

Step-by-step explanation:

cos(x) = - 1 has only 1 solution in the required interval , that is

x = [tex]cos^{-1}[/tex] (- 1) = 180°

Consider the hypotheses shown below. Given that xˉ=49,σ=11,n=32,α=0.10, complete parts a and b. H0​:μ≤47H1​:μ>47​ a. What conclusion should be drawn? b. Determine the p-value for this test. a. The z-test statistic is (Round to two decimal places as needed.) a. The z-test statistic is (Round to two decimal places as needed.) The critical z-score(s) is(are) (Round to two decimal places as needed. Use a comma to separate answers as needed.) Because the test statistic the null hypothesis. b. The p-value is (Round to three decimal places as needed.)

Answers

The conclusions are as follows:

a. We reject the null hypothesis.

b. The p-value for this test is approximately 0.001.

To complete parts a and b, we can follow the steps for hypothesis testing.

a. The z-test statistic can be calculated using the formula:

z = ([tex]\bar{X}[/tex] - μ) / (σ / √n)

Given:

[tex]\bar{X}[/tex] = 49

σ = 11

n = 32

Substituting these values into the formula, we get:

z = (49 - 47) / (11 / √32) ≈ 2.90

The z-test statistic is approximately 2.90 (rounded to two decimal places).

To determine the critical z-score(s), we need to find the z-value that corresponds to the significance level α = 0.10. Since the alternative hypothesis is one-sided (μ > 47), we are performing a right-tailed test.

Using a standard normal distribution table or a calculator, the critical z-score for a right-tailed test at α = 0.10 is approximately 1.28 (rounded to two decimal places).

Because the test statistic z = 2.90 is greater than the critical z-score of 1.28, we reject the null hypothesis.

b. The p-value represents the probability of obtaining a test statistic as extreme as the observed value (or more extreme) assuming the null hypothesis is true. Since the alternative hypothesis is μ > 47, we are looking for the probability in the right tail of the distribution.

Using a standard normal distribution table or a calculator, we can find the p-value corresponding to the z-test statistic z = 2.90. The p-value is the area under the curve to the right of 2.90.

The p-value is approximately 0.001 (rounded to three decimal places).

Therefore, the conclusions are as follows:

a. We reject the null hypothesis.

b. The p-value for this test is approximately 0.001.

Learn more about Z-score here

https://brainly.com/question/31871890

#SPJ4

6(y - 2) = -18

What is y?

Answers

Answer:

-1

Step-by-step explanation:

6 (y - 2) = -18

6y - 12 = -18

6y = -18 + 12

6y = -6

y = -1

If it takes Laine 4 hours to wash the windows, Leslie 5 hours to wash the same windows, and Lance 6 hours to wash the windows, how long would it take them all working together?
.8 hours
6 hours
1.6 hours
3.2 hours

Answers

1/4+1/5+1/6=1/x

LCM = 60

15/60+12/60+10/60=1/x

37/60=1/x

37x=60

x=60/37=1.62, round =1.6

1.6 HOURS

The number of hours they would take them all working together is around 1.6 hours.

What are ratios and proportions?

A ratio is the divide of the two variables and the lower variable known as the denominator should not be zero.

Given that laine takes 4 hours to complete work

In 1 hour Laine will do 1/4 part of the work.

In 1 hour Leslie will do 1/5 part of the work.

In a 1-hour Lance will do 1/6 part of the work.

so let's suppose all done work by x hours then

1/4 + 1/5 + 1/6 = 1/x

x =1.6 so all together done the work by 1.6 hours.

For more information about ratio and proportion.

https://brainly.com/question/26974513

#SPJ5

What is the surface area of the rectangular prism below
7 7 14

A.496 B.490 C.980 D.248

Answers

Answer:

B.490

Step-by-step explanation:

Correct me if im wrong

Answer: 490

Step-by-step explanation:

Assume that 7 people, including the husband and wife pair, apply for 6 sales positions. People are hired at random. What is the probability that one is hired and one is not?

Answers

The probability that one is hired and one is not is 5/7.

Given that there are 7 people, including the husband and wife pair, apply for 6 sales positions. People are hired at random.The probability that one is hired and one is not is obtained as follows:

Total number of ways to choose 6 people out of 7 is given by, `n(S) = 7C6 = 7`

The number of ways in which the husband and wife pair will be selected and 4 other people will be selected out of remaining 5 is given by, `n(E) = 5C4 = 5`

Therefore, the probability that one is hired and one is not can be expressed as:

Probability = n(E) / n(S)

Probability = 5/7

To know more about probability:

https://brainly.com/question/31828911

#SPJ11

There are 7 people for 6 positions. We know that the husband and wife pair both want a job, so we can count them as a single "unit" for this calculation. So there are effectively 6 people for 6 positions.

The required probability is 2/3.

There are two possible outcomes for the husband-wife pair:

Either both are hired or both are not hired. For the probability that one is hired and one is not:

Find the probability that the husband-wife pair are hired and subtract that from 1 (to get the probability that one is hired and one is not). The probability that the husband-wife pair are hired is:

[tex]\frac {\binom{5}{4}}{\binom{6}{4}} = \frac{5}{15}[/tex]

[tex]= \frac{1}{3}[/tex]

So the probability that one is hired and one is not is:

[tex]1 - \frac{1}{3} = \frac{2}{3}[/tex]

The required probability is 2/3.

To know more about probability visit

https://brainly.com/question/32004014

#SPJ11

Other Questions
Which version best uses a variety of sentence structures to enhance the flow and writing style of a story? O A. The seas were rough. The ship sailed on. The passengers in their bunks held on tightly. Some of them felt sick. They tried to hide it. They all yearned for the storm to end. But the waves went on crashing. The captain held the ship to a steady course.O B. Though the seas were rough, the ship sailed on, and the passengers in their bunks held on tightly. While some of them felt sick, they tried to hide it, and they all yearned for the storm to end. As the waves went on crashing, the captain steered the ship, and it sailed a steady course. O C Though the seas were rough, the ship sailed on. The passengers in their bunks held on tightly. Some of them felt sick, though they tried to hide it, and they all yearned for the storm to end. But the waves went on crashing, and the captain held the ship to a steady course,O D The seas were rough, yet the ship sailed on. The passengers in their bunks held on tightly, but some of them felt sick. They tried to hide it, and they all yearned for the storm to end. But the waves went on crashing, and the captain held the ship to a steady course. Describe how the following factors might affect a person relearning of speech and mobility regarding her age. In "The Destructors" the boys are members of the Wormsley Common Gang. Of the following choices, which shows best the way they operate?A "flat" character is one dimensional. "The Destructors" occurs in November. A character that profits from experience and undergoes a change or development is called.Old Misery was too mean to spend money on his property. HELP ASAP!!! A padlock has a rotary dial of numbers from 0 to 59. The combination is a series of three numbers, each between 0 and 59. If you were to guess random numbers, what is the probability of guessing the combination correctly? On July 1, Year 1, Livingston Corporation, a wholesaler of manufacturing equipment, issued $7,200,000 of 8-year, 11% bonds at a market (effective) interest rate of 12%, receiving cash of $6,836,187. Interest on the bonds is payable semiannually on December 31 and June 30. The fiscal year of the company is the calendar year.Required:1. Journalize the entry to record the amount of cash proceeds from the issuance of the bonds on July 1, Year 1.Year 1 July 1 Cash 309.236 Discount on Bonds Payable 3,690,764 Bonds Payable 46,000,0002. Journalize the entries to record the following: A. The first semiannual interest payment on December 31, Year 1, and the amortization of the bond discount, using the straight-line method.B. The interest payment on June 30, Year 2, and the amortization of the bond discount, using the interest method. 3. Determine the total interest expense for Year 1. You have a green house. The green house is made completely of glass, exceptfor the door. The entire building is 15 feet tall. The height of the vertical walls is 10 ft. Thegreen house is 20 ft long (on side with door) and 16 feet wide. The triangles that make up theroof are isosceles triangles (both sides are equal and height is measured at the middle of the base).The door is 8 feet wide and 7 feet tall. Answer each of the following questions about your greenhouse.1. How much glass is used to make this greenhouse? as you drive from the equator toward the north pole, how would the altitude of the celestial north pole star change? Will give BRAINLIEST ok last one help meee A recipe for fruit punch uses 4 ounces of fruit mix tomake 6 glasses of punch. How many glasses of thispunch can be made from 26 ounces of fruit mix?A) 24 B) 32 C) 36D) 48 Which of the following protocols are shown as appearing (i.e., are listed in the Wireshark "protocol" column) in your trace file: TCP, QUIC, HTTP, DNS, UDP, TLSv1.2? which of the following industries is most likely to exhibit the characteristic of free entry? group of answer choices a) nuclear power. b) municipal water and sewer. c) dairy farming. d) airport security. Please tell me how to round 14,780 Correct this sentence: Among the many reasons for his defeat in the election was his arrogant assumption that his constituents were incapable of understanding economic conditions, and his unwarranted attack on his chief opponent.-Was his arrogant assumption that his constituents were incapable of understand economic conditions-Were his arrogant assumption that his constituents were incapable of understanding economic conditions-Were his arrogant assumptions that his constituents were incapable of understanding economical conditions-Was the arrogant assumptions that his constituents was incapable of understanding economic conditions A pet store receives 7 boxes of car food. Each box has 48 cans. The store wants to put the cans in equal stacks of 8 cans. Draw a bar model to help you find how many stacks can be formed (6 marks) Janet has the production function z = (x, y) = 2x +y where x and y are inputs and z is the output. Janets productionfunction is strictly concave and in Of the 100 gymnasts at a meet, 34 are boys. Write a decimal for the part of thegymnasts that are girls. determine the edge connectivity and the vertex connectivity of each graph. You have been looking to buy a pair of shoes and notice that on Saturday they are marked down 20% from the original price. They are still too expensive! On Tuesday the shoes are marked down with an additional 25% off the price from Saturday. They are now $63. (a) What was the original price? (b) How much did you save? (c) What fraction of the original price did you spend? Can someone please help me with 1-9